Jump to content

Different components of angular momentum do not commute


Prometheus

Recommended Posts

I'm having a little trouble with some commutational relations between different components of the angular momentum operators. I'm OK up to:

 

[math][\hat{L_x},\hat{L_y}]=\hat{Y}\hat{P_z}\hat{Z}\hat{P_x} - \hat{Z}\hat{P_x}\hat{Y}\hat{P_z} + \hat{Z}\hat{P_y}\hat{X}\hat{P_z} - \hat{X}\hat{P_z}\hat{Z}\hat{P_y}[/math]

 

and the apparently this is equal to

 

[math]\hat{Y}\hat{P_x}(\hat{P_z}\hat{Z} - \hat{Z}\hat{P_z}) + \hat{X}\hat{P_y}(\hat{Z}\hat{P_z} - \hat{P_z}\hat{Z})[/math]

 

I don't understand this 'factoring out' of operators. I thought it was a typo in my notes and the answer is:

 

[math]\hat{Y}\hat{P_z}(\hat{Z}\hat{P_z} - \hat{Z}\hat{P_z}) + \hat{X}\hat{P_y}(\hat{Z}\hat{P_z} - \hat{Z}\hat{P_z})[/math]

 

but then the rest of the derivation doesn't work.

 

I think i'm missing some property of commutators, but can't see what it is.

 

Help appreciated.

Link to comment
Share on other sites

This may help.

https://www.google.ca/url?sa=t&source=web&rct=j&url=http://www2.ph.ed.ac.uk/~ldeldebb/docs/QM/lect8.pdf&ved=0ahUKEwiw-8a6zKnRAhVcVWMKHQMzC50QFggvMAQ&usg=AFQjCNHaBacLpJQ-41rXS0L58bK_m3Y3BA&sig2=p0G0kk7RP3Dyd8tlwjGq7w

http://farside.ph.utexas.edu/teaching/qmech/Quantum/node71.html

Actually this article may be better in details.

https://www.google.ca/url?sa=t&source=web&rct=j&url=https://ocw.mit.edu/courses/physics/8-05-quantum-physics-ii-fall-2013/lecture-notes/MIT8_05F13_Chap_09.pdf&ved=0ahUKEwitj8nH0KnRAhUC7GMKHYxoCWoQFghOMA8&usg=AFQjCNHiSV17-hjfnjrbqPgfk1uzd8Jmqw&sig2=qNjM2E10sGt4RPiKRgWz4A

remember

[latex][\hat{L_x},\hat{L_y}]=i\hbar \hat{L_z}[/latex],

[latex][\hat{L_y},\hat{L_z}]=i\hbar \hat{L_x}[/latex]

[latex][\hat{L_z},\hat{L_x}]=i\hbar \hat{L_y}[/latex]

 

these are components of a vector on a cartesian coordinate system


[latex]L_x=\frac{\hbar}{\sqrt{2}}\begin{pmatrix}0&1&0\\1&0&1\\0&1&0\end{pmatrix}[/latex]
[latex]L_y=\frac{\hbar}{\sqrt{2i}}\begin{pmatrix}0&1&0\\-1&0&-1\\0&-1&0\end{pmatrix}[/latex]
[latex]L_z=\hbar\begin{pmatrix}1&0&0\\0&0&0\\0&0&-1\end{pmatrix}[/latex]

[latex]\langle\psi|L_x|\psi\rangle=(\psi_1^*,\psi_2^*,\psi_3^*)\frac{\hbar}{\sqrt{2}}\begin{pmatrix}0&1&0\\1&0&1\\0&1&0\end{pmatrix}\begin{pmatrix}\psi_1\\ \psi_2\\ \psi_3\end{pmatrix}[/latex]
[latex]=\frac{\sqrt{\hbar}}{2}(\psi_1^*\psi_2^*\psi_3^*)\begin{pmatrix}\psi_2\\ \psi_1+\psi_3\\\psi_2\end{pmatrix}[/latex]
[latex]=\frac{\sqrt{\hbar}}{2}(\psi_1^*\psi_2\psi_2^*(\psi_1+\psi_2)+\psi_3^*\psi_2)[/latex]
I'm curious how much work have you done with tensors? this seems to be where your having difficulties. Though I could be wrong on that



Though if your missing the details below which I should have started with the above makes more sense :P

[latex]\overrightarrow{L}=\overrightarrow {r}*\overrightarrow{p}=\begin{pmatrix}i&j&k\\x&y&z\\p_x&p_y&p_z\end{pmatrix}=i(yp_z-zp_y)+j(zp_x-xp_z)+k (xp_y-yp_x)[/latex]

 

the magnitude of each component being

[latex]L_x=yp_z-zp_y[/latex]

[latex]L_y=yp_x-zp_z[/latex]

[latex]L_z=yp_y-zp_x[/latex]

 

hope this helps

Edited by Mordred
Link to comment
Share on other sites

edit bad post

I'm having a little trouble with some commutational relations between different components of the angular momentum operators. I'm OK up to:

 

[math][\hat{L_x},\hat{L_y}]=\hat{Y}\hat{P_z}\hat{Z}\hat{P_x} - \hat{Z}\hat{P_x}\hat{Y}\hat{P_z} + \hat{Z}\hat{P_y}\hat{X}\hat{P_z} - \hat{X}\hat{P_z}\hat{Z}\hat{P_y}[/math]

 

and the apparently this is equal to

 

[math]\hat{Y}\hat{P_x}(\hat{P_z}\hat{Z} - \hat{Z}\hat{P_z}) + \hat{X}\hat{P_y}(\hat{Z}\hat{P_z} - \hat{P_z}\hat{Z})[/math]

 

I don't understand this 'factoring out' of operators. I thought it was a typo in my notes and the answer is:

 

[math]\hat{Y}\hat{P_z}(\hat{Z}\hat{P_z} - \hat{Z}\hat{P_z}) + \hat{X}\hat{P_y}(\hat{Z}\hat{P_z} - \hat{Z}\hat{P_z})[/math]

 

but then the rest of the derivation doesn't work.

 

I think i'm missing some property of commutators, but can't see what it is.

 

Help appreciated.

Just re-read this let me think about it.

Edited by Mordred
Link to comment
Share on other sites

I'm having a little trouble with some commutational relations between different components of the angular momentum operators. I'm OK up to:

 

[math][\hat{L_x},\hat{L_y}]=\hat{Y}\hat{P_z}\hat{Z}\hat{P_x} - \hat{Z}\hat{P_x}\hat{Y}\hat{P_z} + \hat{Z}\hat{P_y}\hat{X}\hat{P_z} - \hat{X}\hat{P_z}\hat{Z}\hat{P_y}[/math]

 

and the apparently this is equal to

 

[math]\hat{Y}\hat{P_x}(\hat{P_z}\hat{Z} - \hat{Z}\hat{P_z}) + \hat{X}\hat{P_y}(\hat{Z}\hat{P_z} - \hat{P_z}\hat{Z})[/math]

 

I don't understand this 'factoring out' of operators. I thought it was a typo in my notes and the answer is:

 

[math]\hat{Y}\hat{P_z}(\hat{Z}\hat{P_z} - \hat{Z}\hat{P_z}) + \hat{X}\hat{P_y}(\hat{Z}\hat{P_z} - \hat{Z}\hat{P_z})[/math]

 

but then the rest of the derivation doesn't work.

 

I think i'm missing some property of commutators, but can't see what it is.

 

Help appreciated.

 

 

Remember some assumptions on the commutation relations of position and momentum - position and momentum operators with different indices (e.g. [math]\hat{Y},\hat{P_z}[/math]) commute, while ones with the same index (e.g. [math]\hat{Z},\hat{P_z}[/math]) do not. In your expression, the only terms with common indices deal with Z, so you can feel free to move the others in any way you want - including, as here, to the front. So when you do that for the first term, you get [math]\hat{Y}\hat{P_z}\hat{Z}\hat{P_x} = \hat{Y}\hat{P_x}\hat{P_z}\hat{Z}[/math]. However, you can't then switch [math]\hat{P_z}[/math] and [math]\hat{Z}[/math], as they do not commute - so this is the simplest you can get. Similarly, the second term becomes [math]\hat{P_x}\hat{Y}\hat{Z}\hat{P_z}[/math].

 

Does that help?

Link to comment
Share on other sites

I'm curious how much work have you done with tensors? this seems to be where your having difficulties. Though I could be wrong on that

 

Absolutely none, and it's not coming up either.

 

 

Does that help?

 

Ah, yes. I did know this, but didn't apply it. I'll try later and let you know, but i think i'll be OK now.

 

Yep, that worked a treat, thanks.

Edited by Prometheus
Link to comment
Share on other sites

k good glad you got it. By the time I reread the OP it was too late to think clearly lol. Glad uncool covered it.

 

Good to know on the matrix/tensors not being on your curriculum. It will help better constrain future replies

Edited by Mordred
Link to comment
Share on other sites

Create an account or sign in to comment

You need to be a member in order to leave a comment

Create an account

Sign up for a new account in our community. It's easy!

Register a new account

Sign in

Already have an account? Sign in here.

Sign In Now
×
×
  • Create New...

Important Information

We have placed cookies on your device to help make this website better. You can adjust your cookie settings, otherwise we'll assume you're okay to continue.